Circulation of a 3d vector field

Click For Summary
The discussion focuses on demonstrating that the circulation of the vector field v(x,y,z)=(-h(z)y,h(z)x,g(z)) around a closed curve C in the horizontal plane z=z0 depends solely on the area enclosed by C and the function h evaluated at z0. Participants suggest starting by parameterizing the curve C, for example using [x(t),y(t),z0] for t in [0,1]. The next step involves writing the line integral that defines circulation and factoring out constants. This approach leads to recognizing a formula for area in the remaining expression. Ultimately, the solution hinges on understanding the relationship between circulation, area, and the function h(z).
cummings12332
Messages
38
Reaction score
0

Homework Statement


consider the vector field v(x,y,z)=(-h(z)y,h(z)x,g(z)) wherer h:R->R and g:R—>R are differentiable .Let C be a closed curve in the horizontal plane z=z0.show that the circulation of v around C depends only on the area of the reion enclosed by C in the given plane and h(Z0)


The Attempt at a Solution


i know it is simple,but i don't know how to star it. should i parameterize the curve c ? but how? could someone give me some details hints?
 
Physics news on Phys.org
cummings12332 said:

Homework Statement


consider the vector field v(x,y,z)=(-h(z)y,h(z)x,g(z)) wherer h:R->R and g:R—>R are differentiable .Let C be a closed curve in the horizontal plane z=z0.show that the circulation of v around C depends only on the area of the reion enclosed by C in the given plane and h(Z0)


The Attempt at a Solution


i know it is simple,but i don't know how to star it. should i parameterize the curve c ? but how? could someone give me some details hints?

Parameterize any way you like, say [x(t),y(t),z0], for t in [0,1]. Write down the line integral that defines "circulation" around your curve. Factor out constants. Do you recognize a formula for area in what's left?
 
Question: A clock's minute hand has length 4 and its hour hand has length 3. What is the distance between the tips at the moment when it is increasing most rapidly?(Putnam Exam Question) Answer: Making assumption that both the hands moves at constant angular velocities, the answer is ## \sqrt{7} .## But don't you think this assumption is somewhat doubtful and wrong?

Similar threads

  • · Replies 8 ·
Replies
8
Views
2K
  • · Replies 1 ·
Replies
1
Views
1K
  • · Replies 1 ·
Replies
1
Views
2K
Replies
3
Views
2K
  • · Replies 8 ·
Replies
8
Views
2K
  • · Replies 8 ·
Replies
8
Views
2K
  • · Replies 21 ·
Replies
21
Views
4K
  • · Replies 13 ·
Replies
13
Views
1K
  • · Replies 3 ·
Replies
3
Views
2K
  • · Replies 2 ·
Replies
2
Views
2K